Proof regarding quasi-solutions












2












$begingroup$


I'm almost done proving a theorem regarding quasi-solutions, which I'm stating here:



$Let A:Xrightarrow Y be a compact linear operator and let rho>0. Then for each fin Y there exists a unique element varphi_0in X with ||varphi_0||leqrho satisfying$



$$||Avarphi_0-f||leq||Avarphi-f||$$



$for all varphiin X with ||varphi||leqrho.$



Here $X$ and $Y$ are Hilbert spaces. The part of the proof I'm struggling with is the following (uniqueness): We're considering the set $V:={Avarphi | ||varphi||leqrho}$ since for $varphi_0$ to be a quasi-solution with the constraint $rho$, then it has to be the best approximation to the set $V$.



I've already shown that $V$ is a convex set. The problem now is to show that there in the Hilbert space $Y$ exist at most one best approximation to $f$ with respect to the set $V$.



Here's my idea on how to do it (showing uniqueness of the quasi-solution):



If we consider two points $v_0,v_1in V$ in which we assume they both are the best approximation to some element $yin Y$, i.e., $||y-v_0||=||y-v_1||$. Now, we should consider the line segment between $v_0$ and $v_1$ (we know this is contained in $V$ since it is convex) and consider the distance from that to $y$, i.e., $||y-(tv_0+(1-t)v_1)||$ for $tin[0,1]$. My idea is now to show that one can find a unique $t$ for which the distance is the smallest. This will contradict the assumption that both $v_0$ and $v_1$ were the best approximations to $y$ since we've now found a new one.



Does this seem correct or am I missing something?



Thanks in advance!










share|cite|improve this question











$endgroup$












  • $begingroup$
    you should state at the beginning, what spaces $X$ (and $Y$) are (probably Hilbert spaces). Then it is unclear whether your question is also about existence of $varphi_0$ or if you are only interested in uniqueness.
    $endgroup$
    – supinf
    Jan 8 at 9:47










  • $begingroup$
    Sorry, both $X$ and $Y$ are Hilbert spaces. I have shown existence of the quasi-solution so I'm only interested in uniqueness.
    $endgroup$
    – James
    Jan 8 at 9:50
















2












$begingroup$


I'm almost done proving a theorem regarding quasi-solutions, which I'm stating here:



$Let A:Xrightarrow Y be a compact linear operator and let rho>0. Then for each fin Y there exists a unique element varphi_0in X with ||varphi_0||leqrho satisfying$



$$||Avarphi_0-f||leq||Avarphi-f||$$



$for all varphiin X with ||varphi||leqrho.$



Here $X$ and $Y$ are Hilbert spaces. The part of the proof I'm struggling with is the following (uniqueness): We're considering the set $V:={Avarphi | ||varphi||leqrho}$ since for $varphi_0$ to be a quasi-solution with the constraint $rho$, then it has to be the best approximation to the set $V$.



I've already shown that $V$ is a convex set. The problem now is to show that there in the Hilbert space $Y$ exist at most one best approximation to $f$ with respect to the set $V$.



Here's my idea on how to do it (showing uniqueness of the quasi-solution):



If we consider two points $v_0,v_1in V$ in which we assume they both are the best approximation to some element $yin Y$, i.e., $||y-v_0||=||y-v_1||$. Now, we should consider the line segment between $v_0$ and $v_1$ (we know this is contained in $V$ since it is convex) and consider the distance from that to $y$, i.e., $||y-(tv_0+(1-t)v_1)||$ for $tin[0,1]$. My idea is now to show that one can find a unique $t$ for which the distance is the smallest. This will contradict the assumption that both $v_0$ and $v_1$ were the best approximations to $y$ since we've now found a new one.



Does this seem correct or am I missing something?



Thanks in advance!










share|cite|improve this question











$endgroup$












  • $begingroup$
    you should state at the beginning, what spaces $X$ (and $Y$) are (probably Hilbert spaces). Then it is unclear whether your question is also about existence of $varphi_0$ or if you are only interested in uniqueness.
    $endgroup$
    – supinf
    Jan 8 at 9:47










  • $begingroup$
    Sorry, both $X$ and $Y$ are Hilbert spaces. I have shown existence of the quasi-solution so I'm only interested in uniqueness.
    $endgroup$
    – James
    Jan 8 at 9:50














2












2








2


1



$begingroup$


I'm almost done proving a theorem regarding quasi-solutions, which I'm stating here:



$Let A:Xrightarrow Y be a compact linear operator and let rho>0. Then for each fin Y there exists a unique element varphi_0in X with ||varphi_0||leqrho satisfying$



$$||Avarphi_0-f||leq||Avarphi-f||$$



$for all varphiin X with ||varphi||leqrho.$



Here $X$ and $Y$ are Hilbert spaces. The part of the proof I'm struggling with is the following (uniqueness): We're considering the set $V:={Avarphi | ||varphi||leqrho}$ since for $varphi_0$ to be a quasi-solution with the constraint $rho$, then it has to be the best approximation to the set $V$.



I've already shown that $V$ is a convex set. The problem now is to show that there in the Hilbert space $Y$ exist at most one best approximation to $f$ with respect to the set $V$.



Here's my idea on how to do it (showing uniqueness of the quasi-solution):



If we consider two points $v_0,v_1in V$ in which we assume they both are the best approximation to some element $yin Y$, i.e., $||y-v_0||=||y-v_1||$. Now, we should consider the line segment between $v_0$ and $v_1$ (we know this is contained in $V$ since it is convex) and consider the distance from that to $y$, i.e., $||y-(tv_0+(1-t)v_1)||$ for $tin[0,1]$. My idea is now to show that one can find a unique $t$ for which the distance is the smallest. This will contradict the assumption that both $v_0$ and $v_1$ were the best approximations to $y$ since we've now found a new one.



Does this seem correct or am I missing something?



Thanks in advance!










share|cite|improve this question











$endgroup$




I'm almost done proving a theorem regarding quasi-solutions, which I'm stating here:



$Let A:Xrightarrow Y be a compact linear operator and let rho>0. Then for each fin Y there exists a unique element varphi_0in X with ||varphi_0||leqrho satisfying$



$$||Avarphi_0-f||leq||Avarphi-f||$$



$for all varphiin X with ||varphi||leqrho.$



Here $X$ and $Y$ are Hilbert spaces. The part of the proof I'm struggling with is the following (uniqueness): We're considering the set $V:={Avarphi | ||varphi||leqrho}$ since for $varphi_0$ to be a quasi-solution with the constraint $rho$, then it has to be the best approximation to the set $V$.



I've already shown that $V$ is a convex set. The problem now is to show that there in the Hilbert space $Y$ exist at most one best approximation to $f$ with respect to the set $V$.



Here's my idea on how to do it (showing uniqueness of the quasi-solution):



If we consider two points $v_0,v_1in V$ in which we assume they both are the best approximation to some element $yin Y$, i.e., $||y-v_0||=||y-v_1||$. Now, we should consider the line segment between $v_0$ and $v_1$ (we know this is contained in $V$ since it is convex) and consider the distance from that to $y$, i.e., $||y-(tv_0+(1-t)v_1)||$ for $tin[0,1]$. My idea is now to show that one can find a unique $t$ for which the distance is the smallest. This will contradict the assumption that both $v_0$ and $v_1$ were the best approximations to $y$ since we've now found a new one.



Does this seem correct or am I missing something?



Thanks in advance!







real-analysis functional-analysis






share|cite|improve this question















share|cite|improve this question













share|cite|improve this question




share|cite|improve this question








edited Jan 8 at 9:51







James

















asked Jan 8 at 9:37









JamesJames

1267




1267












  • $begingroup$
    you should state at the beginning, what spaces $X$ (and $Y$) are (probably Hilbert spaces). Then it is unclear whether your question is also about existence of $varphi_0$ or if you are only interested in uniqueness.
    $endgroup$
    – supinf
    Jan 8 at 9:47










  • $begingroup$
    Sorry, both $X$ and $Y$ are Hilbert spaces. I have shown existence of the quasi-solution so I'm only interested in uniqueness.
    $endgroup$
    – James
    Jan 8 at 9:50


















  • $begingroup$
    you should state at the beginning, what spaces $X$ (and $Y$) are (probably Hilbert spaces). Then it is unclear whether your question is also about existence of $varphi_0$ or if you are only interested in uniqueness.
    $endgroup$
    – supinf
    Jan 8 at 9:47










  • $begingroup$
    Sorry, both $X$ and $Y$ are Hilbert spaces. I have shown existence of the quasi-solution so I'm only interested in uniqueness.
    $endgroup$
    – James
    Jan 8 at 9:50
















$begingroup$
you should state at the beginning, what spaces $X$ (and $Y$) are (probably Hilbert spaces). Then it is unclear whether your question is also about existence of $varphi_0$ or if you are only interested in uniqueness.
$endgroup$
– supinf
Jan 8 at 9:47




$begingroup$
you should state at the beginning, what spaces $X$ (and $Y$) are (probably Hilbert spaces). Then it is unclear whether your question is also about existence of $varphi_0$ or if you are only interested in uniqueness.
$endgroup$
– supinf
Jan 8 at 9:47












$begingroup$
Sorry, both $X$ and $Y$ are Hilbert spaces. I have shown existence of the quasi-solution so I'm only interested in uniqueness.
$endgroup$
– James
Jan 8 at 9:50




$begingroup$
Sorry, both $X$ and $Y$ are Hilbert spaces. I have shown existence of the quasi-solution so I'm only interested in uniqueness.
$endgroup$
– James
Jan 8 at 9:50










1 Answer
1






active

oldest

votes


















1












$begingroup$

Your idea to consider the line between $v_1,v_0$ is a good start.
However, it is often enough to only consider the central point $v_2:=tfrac12(v_0+v_1)$ to get a contradiction.



Here is a sketch on how to do this:



Using the parallelogram law, we have
$$
2|y-v_0|^2+2|y-v_1|^2 = |2y-v_0-v_1|^2+|v_0-v_1|^2
=4|y-v_2|^2+|v_0-v_1|^2.
$$

Now if you assume that $v_0neq v_1$ then it follows that $v_2$ is a better solution than $v_1$ or $v_0$.






share|cite|improve this answer









$endgroup$













  • $begingroup$
    Thank you for the help! I've tried it myself and I got the same as you with the parallelogram law. This is just to see if I understand why you can argue $v_2$ is a better choice than $v_0$ or $v_1$: We assumed that $||y-v_0||=||y-v_1||$ and $v_0neq v_1$, which means by the parallelogram law we have $$4||y-v_0||^2=4||y-v_2||^2+||v_0-v_1||$$ which means that $v_2$ is a better approximation, due to $||v_0-v_1||^2$ term, right?
    $endgroup$
    – James
    Jan 8 at 10:38












  • $begingroup$
    Or one could write the LHS of the parallelogram law with $v_1$, but the idea is that it shows that 4 times the distance between $y$ and $v_0$ or $v_1$ yields 4 times the distance of $y$ and $v_2$ plus an additional distance, namely the distance between $v_0$ and $v_1$, which means $v_2$ must approximate $y$ better.
    $endgroup$
    – James
    Jan 8 at 10:43






  • 1




    $begingroup$
    yes, that is correct
    $endgroup$
    – supinf
    Jan 8 at 11:19











Your Answer





StackExchange.ifUsing("editor", function () {
return StackExchange.using("mathjaxEditing", function () {
StackExchange.MarkdownEditor.creationCallbacks.add(function (editor, postfix) {
StackExchange.mathjaxEditing.prepareWmdForMathJax(editor, postfix, [["$", "$"], ["\\(","\\)"]]);
});
});
}, "mathjax-editing");

StackExchange.ready(function() {
var channelOptions = {
tags: "".split(" "),
id: "69"
};
initTagRenderer("".split(" "), "".split(" "), channelOptions);

StackExchange.using("externalEditor", function() {
// Have to fire editor after snippets, if snippets enabled
if (StackExchange.settings.snippets.snippetsEnabled) {
StackExchange.using("snippets", function() {
createEditor();
});
}
else {
createEditor();
}
});

function createEditor() {
StackExchange.prepareEditor({
heartbeatType: 'answer',
autoActivateHeartbeat: false,
convertImagesToLinks: true,
noModals: true,
showLowRepImageUploadWarning: true,
reputationToPostImages: 10,
bindNavPrevention: true,
postfix: "",
imageUploader: {
brandingHtml: "Powered by u003ca class="icon-imgur-white" href="https://imgur.com/"u003eu003c/au003e",
contentPolicyHtml: "User contributions licensed under u003ca href="https://creativecommons.org/licenses/by-sa/3.0/"u003ecc by-sa 3.0 with attribution requiredu003c/au003e u003ca href="https://stackoverflow.com/legal/content-policy"u003e(content policy)u003c/au003e",
allowUrls: true
},
noCode: true, onDemand: true,
discardSelector: ".discard-answer"
,immediatelyShowMarkdownHelp:true
});


}
});














draft saved

draft discarded


















StackExchange.ready(
function () {
StackExchange.openid.initPostLogin('.new-post-login', 'https%3a%2f%2fmath.stackexchange.com%2fquestions%2f3065974%2fproof-regarding-quasi-solutions%23new-answer', 'question_page');
}
);

Post as a guest















Required, but never shown

























1 Answer
1






active

oldest

votes








1 Answer
1






active

oldest

votes









active

oldest

votes






active

oldest

votes









1












$begingroup$

Your idea to consider the line between $v_1,v_0$ is a good start.
However, it is often enough to only consider the central point $v_2:=tfrac12(v_0+v_1)$ to get a contradiction.



Here is a sketch on how to do this:



Using the parallelogram law, we have
$$
2|y-v_0|^2+2|y-v_1|^2 = |2y-v_0-v_1|^2+|v_0-v_1|^2
=4|y-v_2|^2+|v_0-v_1|^2.
$$

Now if you assume that $v_0neq v_1$ then it follows that $v_2$ is a better solution than $v_1$ or $v_0$.






share|cite|improve this answer









$endgroup$













  • $begingroup$
    Thank you for the help! I've tried it myself and I got the same as you with the parallelogram law. This is just to see if I understand why you can argue $v_2$ is a better choice than $v_0$ or $v_1$: We assumed that $||y-v_0||=||y-v_1||$ and $v_0neq v_1$, which means by the parallelogram law we have $$4||y-v_0||^2=4||y-v_2||^2+||v_0-v_1||$$ which means that $v_2$ is a better approximation, due to $||v_0-v_1||^2$ term, right?
    $endgroup$
    – James
    Jan 8 at 10:38












  • $begingroup$
    Or one could write the LHS of the parallelogram law with $v_1$, but the idea is that it shows that 4 times the distance between $y$ and $v_0$ or $v_1$ yields 4 times the distance of $y$ and $v_2$ plus an additional distance, namely the distance between $v_0$ and $v_1$, which means $v_2$ must approximate $y$ better.
    $endgroup$
    – James
    Jan 8 at 10:43






  • 1




    $begingroup$
    yes, that is correct
    $endgroup$
    – supinf
    Jan 8 at 11:19
















1












$begingroup$

Your idea to consider the line between $v_1,v_0$ is a good start.
However, it is often enough to only consider the central point $v_2:=tfrac12(v_0+v_1)$ to get a contradiction.



Here is a sketch on how to do this:



Using the parallelogram law, we have
$$
2|y-v_0|^2+2|y-v_1|^2 = |2y-v_0-v_1|^2+|v_0-v_1|^2
=4|y-v_2|^2+|v_0-v_1|^2.
$$

Now if you assume that $v_0neq v_1$ then it follows that $v_2$ is a better solution than $v_1$ or $v_0$.






share|cite|improve this answer









$endgroup$













  • $begingroup$
    Thank you for the help! I've tried it myself and I got the same as you with the parallelogram law. This is just to see if I understand why you can argue $v_2$ is a better choice than $v_0$ or $v_1$: We assumed that $||y-v_0||=||y-v_1||$ and $v_0neq v_1$, which means by the parallelogram law we have $$4||y-v_0||^2=4||y-v_2||^2+||v_0-v_1||$$ which means that $v_2$ is a better approximation, due to $||v_0-v_1||^2$ term, right?
    $endgroup$
    – James
    Jan 8 at 10:38












  • $begingroup$
    Or one could write the LHS of the parallelogram law with $v_1$, but the idea is that it shows that 4 times the distance between $y$ and $v_0$ or $v_1$ yields 4 times the distance of $y$ and $v_2$ plus an additional distance, namely the distance between $v_0$ and $v_1$, which means $v_2$ must approximate $y$ better.
    $endgroup$
    – James
    Jan 8 at 10:43






  • 1




    $begingroup$
    yes, that is correct
    $endgroup$
    – supinf
    Jan 8 at 11:19














1












1








1





$begingroup$

Your idea to consider the line between $v_1,v_0$ is a good start.
However, it is often enough to only consider the central point $v_2:=tfrac12(v_0+v_1)$ to get a contradiction.



Here is a sketch on how to do this:



Using the parallelogram law, we have
$$
2|y-v_0|^2+2|y-v_1|^2 = |2y-v_0-v_1|^2+|v_0-v_1|^2
=4|y-v_2|^2+|v_0-v_1|^2.
$$

Now if you assume that $v_0neq v_1$ then it follows that $v_2$ is a better solution than $v_1$ or $v_0$.






share|cite|improve this answer









$endgroup$



Your idea to consider the line between $v_1,v_0$ is a good start.
However, it is often enough to only consider the central point $v_2:=tfrac12(v_0+v_1)$ to get a contradiction.



Here is a sketch on how to do this:



Using the parallelogram law, we have
$$
2|y-v_0|^2+2|y-v_1|^2 = |2y-v_0-v_1|^2+|v_0-v_1|^2
=4|y-v_2|^2+|v_0-v_1|^2.
$$

Now if you assume that $v_0neq v_1$ then it follows that $v_2$ is a better solution than $v_1$ or $v_0$.







share|cite|improve this answer












share|cite|improve this answer



share|cite|improve this answer










answered Jan 8 at 10:06









supinfsupinf

6,1391028




6,1391028












  • $begingroup$
    Thank you for the help! I've tried it myself and I got the same as you with the parallelogram law. This is just to see if I understand why you can argue $v_2$ is a better choice than $v_0$ or $v_1$: We assumed that $||y-v_0||=||y-v_1||$ and $v_0neq v_1$, which means by the parallelogram law we have $$4||y-v_0||^2=4||y-v_2||^2+||v_0-v_1||$$ which means that $v_2$ is a better approximation, due to $||v_0-v_1||^2$ term, right?
    $endgroup$
    – James
    Jan 8 at 10:38












  • $begingroup$
    Or one could write the LHS of the parallelogram law with $v_1$, but the idea is that it shows that 4 times the distance between $y$ and $v_0$ or $v_1$ yields 4 times the distance of $y$ and $v_2$ plus an additional distance, namely the distance between $v_0$ and $v_1$, which means $v_2$ must approximate $y$ better.
    $endgroup$
    – James
    Jan 8 at 10:43






  • 1




    $begingroup$
    yes, that is correct
    $endgroup$
    – supinf
    Jan 8 at 11:19


















  • $begingroup$
    Thank you for the help! I've tried it myself and I got the same as you with the parallelogram law. This is just to see if I understand why you can argue $v_2$ is a better choice than $v_0$ or $v_1$: We assumed that $||y-v_0||=||y-v_1||$ and $v_0neq v_1$, which means by the parallelogram law we have $$4||y-v_0||^2=4||y-v_2||^2+||v_0-v_1||$$ which means that $v_2$ is a better approximation, due to $||v_0-v_1||^2$ term, right?
    $endgroup$
    – James
    Jan 8 at 10:38












  • $begingroup$
    Or one could write the LHS of the parallelogram law with $v_1$, but the idea is that it shows that 4 times the distance between $y$ and $v_0$ or $v_1$ yields 4 times the distance of $y$ and $v_2$ plus an additional distance, namely the distance between $v_0$ and $v_1$, which means $v_2$ must approximate $y$ better.
    $endgroup$
    – James
    Jan 8 at 10:43






  • 1




    $begingroup$
    yes, that is correct
    $endgroup$
    – supinf
    Jan 8 at 11:19
















$begingroup$
Thank you for the help! I've tried it myself and I got the same as you with the parallelogram law. This is just to see if I understand why you can argue $v_2$ is a better choice than $v_0$ or $v_1$: We assumed that $||y-v_0||=||y-v_1||$ and $v_0neq v_1$, which means by the parallelogram law we have $$4||y-v_0||^2=4||y-v_2||^2+||v_0-v_1||$$ which means that $v_2$ is a better approximation, due to $||v_0-v_1||^2$ term, right?
$endgroup$
– James
Jan 8 at 10:38






$begingroup$
Thank you for the help! I've tried it myself and I got the same as you with the parallelogram law. This is just to see if I understand why you can argue $v_2$ is a better choice than $v_0$ or $v_1$: We assumed that $||y-v_0||=||y-v_1||$ and $v_0neq v_1$, which means by the parallelogram law we have $$4||y-v_0||^2=4||y-v_2||^2+||v_0-v_1||$$ which means that $v_2$ is a better approximation, due to $||v_0-v_1||^2$ term, right?
$endgroup$
– James
Jan 8 at 10:38














$begingroup$
Or one could write the LHS of the parallelogram law with $v_1$, but the idea is that it shows that 4 times the distance between $y$ and $v_0$ or $v_1$ yields 4 times the distance of $y$ and $v_2$ plus an additional distance, namely the distance between $v_0$ and $v_1$, which means $v_2$ must approximate $y$ better.
$endgroup$
– James
Jan 8 at 10:43




$begingroup$
Or one could write the LHS of the parallelogram law with $v_1$, but the idea is that it shows that 4 times the distance between $y$ and $v_0$ or $v_1$ yields 4 times the distance of $y$ and $v_2$ plus an additional distance, namely the distance between $v_0$ and $v_1$, which means $v_2$ must approximate $y$ better.
$endgroup$
– James
Jan 8 at 10:43




1




1




$begingroup$
yes, that is correct
$endgroup$
– supinf
Jan 8 at 11:19




$begingroup$
yes, that is correct
$endgroup$
– supinf
Jan 8 at 11:19


















draft saved

draft discarded




















































Thanks for contributing an answer to Mathematics Stack Exchange!


  • Please be sure to answer the question. Provide details and share your research!

But avoid



  • Asking for help, clarification, or responding to other answers.

  • Making statements based on opinion; back them up with references or personal experience.


Use MathJax to format equations. MathJax reference.


To learn more, see our tips on writing great answers.




draft saved


draft discarded














StackExchange.ready(
function () {
StackExchange.openid.initPostLogin('.new-post-login', 'https%3a%2f%2fmath.stackexchange.com%2fquestions%2f3065974%2fproof-regarding-quasi-solutions%23new-answer', 'question_page');
}
);

Post as a guest















Required, but never shown





















































Required, but never shown














Required, but never shown












Required, but never shown







Required, but never shown

































Required, but never shown














Required, but never shown












Required, but never shown







Required, but never shown







Popular posts from this blog

Mario Kart Wii

What does “Dominus providebit” mean?

Antonio Litta Visconti Arese